0 Daumen
598 Aufrufe

D2B37F18-A670-493A-9615-90D0C6B48180.jpeg

Text erkannt:

(a) Bestimmen Sie alle \( x \in \mathbb{R} \), für die die Potenzreihe
$$ \sum \limits_{k=1}^{\infty} \frac{2^{k}}{k^{2}} x^{k} $$
konvergiert.
(b) Bestimmen Sie die Taylorreihen der Funktionen
$$ f(x)=\ln \left(\sqrt{\frac{1+x}{1-x}}\right), \quad g(x)=\frac{x}{1+x-2 x^{2}} $$
\( \mathrm{im} \) Entwicklungspunkt \( 0 . \) [Hinweis für \( g \) : Partialbruchzerlegung und geometrische Reihe]

Hallo, Ich bräuchte etwas Hilfe bei einem Matheaufgabe. Wüsste vielleicht jemand wie ich bei a und b vorgehen müsste? Vielen Dank im Voraus ☺️

Avatar von

1 Antwort

0 Daumen
 
Beste Antwort

Aloha :)

zu a) Der Konvergenzradius von \(S(x)\coloneqq\sum\limits_{k=1}^\infty a_k\cdot x^k\) mit \(a_k\coloneqq\frac{2^k}{k^2}\) lautet:$$r=\lim\limits_{k\to\infty}\left|\frac{a_k}{a_{k+1}}\right|=\lim\limits_{k\to\infty}\left|\frac{2^k}{k^2}\,\frac{(k+1)^2}{2^{k+1}}\right|=\lim\limits_{k\to\infty}\left(\frac{2^k}{2^{k+1}}\,\frac{(k+1)^2}{k^2}\right)$$$$\phantom{r}=\lim\limits_{k\to\infty}\frac{1}{2}\left(1+\frac{1}{k}\right)^2=\frac{1}{2}$$Die Potenzreihe konvergiert also sicher für \(|x|<\frac{1}{2}\). Da wir alle \(x\) bestimmen sollen, für die die Potenzreihe konvergiert, müssen wir die beiden Randfälle \(x=\pm\frac{1}{2}\) noch separat prüfen:

$$x=+\frac{1}{2}\implies S(x)=\sum\limits_{k=1}^\infty\frac{2^k}{k^2}\,\left(\frac{1}{2}\right)^k=\sum\limits_{k=1}^\infty\frac{1}{k^2}=\frac{\pi^2}{6}$$$$x=-\frac{1}{2}\implies S(x)=\sum\limits_{k=1}^\infty\frac{2^k}{k^2}\,\left(-\frac{1}{2}\right)^k=\sum\limits_{k=1}^\infty\frac{(-1)^k}{k^2}=-\frac{\pi^2}{12}$$Die beiden Summen an den Randpunkten des Konvergenzradius sind die bekannten Euler-Summen und konvergieren.

Die Potenzreihe konvergiert also für \(x\in\left[-\frac{1}{2}\big|+\frac{1}{2}\right]\).

zu b1) Hier kannst du auf der bekannten Summenformel$$\ln(1+x)=-\sum\limits_{k=1}^\infty\frac{(-x)^k}{k}$$für den Logarithmus aufbauen, dann:

$$f(x)=\ln\left(\sqrt{\frac{1+x}{1-x}}\right)=\frac{1}{2}\ln\left(\frac{1+x}{1-x}\right)=\frac{1}{2}\left(\ln(1+x)-\ln(1-x)\right)$$$$\phantom{f(x)}=\frac{1}{2}\left(-\sum\limits_{k=1}^\infty\frac{(-x)^k}{k}+\sum\limits_{k=1}^\infty\frac{x^k}{k}\right)=\frac{1}{2}\sum\limits_{k=1}^\infty\frac{2x^{2k-1}}{2k-1}=\sum\limits_{k=1}^\infty\frac{x^{2k-1}}{2k-1}$$

zu b2) Hier folgen wir der Bedienungsanleitung und verwenden die geometrische Reihe$$\sum\limits_{k=0}^\infty q^k=\frac{1}{1-q}$$nach der Partialbruchzerlegung:$$g(x)=\frac{x}{1+x-2x^2}=\frac{-1}{3(2x+1)}+\frac{-1}{3(x-1)}=-\frac{1}{3}\,\frac{1}{1+2x}+\frac{1}{3}\,\frac{1}{1-x}$$$$\phantom{g(x)}=-\frac{1}{3}\sum\limits_{k=0}^\infty(-2x)^k+\frac{1}{3}\sum\limits_{k=0}^\infty x^k=\frac{1}{3}\sum\limits_{k=0}^\infty\left(x^k-(-2)^kx^k\right)$$$$\phantom{g(x)}=\frac{1}{3}\sum\limits_{k=0}^\infty\left(1-(-2)^k\right)x^k=\frac{1}{3}\sum\limits_{k=1}^\infty\left(1-(-2)^k\right)x^k$$

Avatar von 148 k 🚀

Dankeschön:)

Ein anderes Problem?

Stell deine Frage

Willkommen bei der Mathelounge! Stell deine Frage einfach und kostenlos

x
Made by a lovely community